–20, - 12,5, 22, . . .
An=
A50=

Answers

Answer 1

Answer:

an = 17n - 46

a50 = 804

Step-by-step explanation:

-29, -12, 5, 22, ...

Subtract each number from the next one.

-12 - (-29) = 17

5 - (-12) = 17

22 - 5 = 17

The common difference is 17. This is an arithmetic sequence which starts with -29, and in which each subsequent value is 17 more than the previous value.

a1 = -29

a2 = -29 + 17

a3 = -29 + 2(17)

a4 = -29 + 3(17)

Notice that for each term, you have -29 and something added to it. What you add to -29 is 17 multiplied by 1 less than the number of the term. For term 1, 1 less than 1 is 0. You add 0 * 17 to -29 and get -29. term 1 is -29. For term 2, 1 less than 2 is 1. You add 1 * 17 to -29 and get -12, etc.

For term n, 1 less than n is n - 1. Add (n - 1) * 17 to -29 to get term n.

an = -29 + 17(n - 1)

This formula can be simplified.

an = -29 + 17n - 17

an = -46 + 17n

an = 17n - 46

a50 = 17(50) - 46

a50 = 804


Related Questions

Find the area of the following shape.

Answers

Answer:

57 units^2

Step-by-step explanation:

First find the area of the triangle on the left

ABC

It has a base AC which is  9 units and a height of 3 units

A = 1/2 bh = 1/2 ( 9) *3 = 27/2 = 13.5

Then find the area of the triangle on the right

DE

It has a base AC which is  6 units and a height of 1 units

A = 1/2 bh = 1/2 ( 6) *1  = 3

Then find the area of the triangle on the top

It has a base AC which is  3 units and a height of 3 units

A = 1/2 bh = 1/2 ( 3) *3  = 9/2 = 4.5

Then find the area of the rectangular region

A = lw = 6*6 = 36

Add them together

13.5+3+4.5+36 =57 units^2

Answer:

Total Area = 57 sq. units

Step-by-step explanation:

will make it simple and short

Total Area = A1 + A2 + A3

A1 = (7 + 6) * 6/2 = 39 sq. units  (area of a trapezoid)

A2 = 1/2 (9 * 3) = 13.5 sq. units (area of a triangle)

A3 = 1/2 (3 * 3) = 4.5 sq. units  (area of a triangle)

Total Area = 39 + 13.5 + 4.5 = 57 sq. units

HELP!!!!!!!!!!!!!!!!

Answers

Answer:

14...

area = 1/2 b h

b= 7-0 = 7 ... 7/2 = 3.5

h = 7-3 = 4

3.5 * 4 = 14

Step-by-step explanation:

Determine if the triangles are congruent.

Answers

Answer:

Answer to number 3:

Yes, because these two are similar triangles for sure so all their respective angles are equal in measure, and two pairs of sides are congruent to each other, so the third one must be congruent too.

Find the measure of each angle indicated. Round to the nearest tenth.

A) 65.20
C) 55.1°
B) 51°
D) 55.70​

Answers

Answer:

51

Step-by-step explanation:

Answer: D) 55.7 degrees approximately

=====================================================

The reference angle has AC = 12.3 as the opposite side and BC = 8.4 as the adjacent side. The tangent ratio ties the opposite and adjacent sides together.

--------

tan(angle) = opposite/adjacent

tan(theta) = AC/BC

tan(theta) = 12.3/8.4

theta = arctan(12.3/8.4)

theta = 55.6697828044967

theta = 55.7 degrees approximately

--------

arctan is the same as inverse tangent which is written as [tex]\tan^{-1}[/tex]

make sure your calculator is in degree mode

If 48% of the students in a certain college are female and there are 1440 female students, what is the total number of students in the college?

Answers

Answer:

3000 students

Step-by-step explanation:

If 48% of the students are female, and there are 1440 female students, we can set up a percentage proportion, assuming x is the total amount of students.

[tex]\frac{1440}{x} = \frac{48}{100}[/tex]

We can use the cross products property to find the value of x.

[tex]1440\cdot100=144000\\\\144000\div48=3000[/tex]

Hope this helped!

What is the value of a in the given figure​

Answers

Answer:

soln

let one side be c

let another side be d

then

40=d(vertically opposite angle)

5a =c(vertically opposite angle)

again

c+90=180(being linear pair)

or, c=180-90

therefore, c=90

finally

let c be a

5a=5×90

=450

Answer:

a = 10

Step-by-step explanation:

CF is a straight line , so the 3 angles on it sum to 180° , then

∠ BOA = 180° - (90 + 40)° = 180° - 130° = 50°

∠ DOE = ∠ BOA ( vertical angles ) , then

5a = 50 ( divide both sides by 5 )

a = 10

Find th slope of the line passing through the points (5,8) and (6,12)

Answers

Answer:

Slope= 4

Step-by-step explanation:

y2-y1/x2-x1

5=x1 8=y1 etc.

12-8/6-5

=4/1

Answer:

The slope of the line is 4.

Step-by-step explanation:

To solve for the slope of the line, use the rise over run formula. The rise over run formula is a measure of how much something changes vertically compared to how much it changes in the horizontal direction. The rise over run formula calculates the difference between the two points in the vertical direction (rise) and then divide it by the difference in the horizontal direction (run). The formula looks like [tex]\frac{rise}{run}[/tex], but in terms of finding a slope, it looks like [tex]\frac{y_{2}-y_{1} }{x_2-x_1}[/tex].

For finding the slope of this line with the points (5,8) and (6,12), start by writing down the information given from the question.

[tex]y_2=12[/tex]

[tex]y_1=8[/tex]

[tex]x_2=6[/tex]

[tex]x_1=5[/tex]

Next, plug in the information given from the question into the formula, and the formula will look like [tex]\frac{12-8}{6-5}[/tex]. Then, simplify the equation, which will look like [tex]\frac{4}{1}=4[/tex]. The final answer will be 4.

Helppp thank you!!!!!

Answers

Answer:

D

Step-by-step explanation:

x is greater than 12:

[tex]x>12 \text{ or } 12<x[/tex]

and x is less than or equal to 3 times the value of y:

[tex]12<x\leq 3y[/tex]

We can divide everything by 3:

[tex]4<\frac{x}{3} \leq y[/tex]

which one is correct?

Answers

Answer:

[tex] (x+4)^2=4[/tex]

Step-by-step explanation:

[tex]x^2+8x+12=0\\

\implies (x^2+8x+16)+12=16\\

\implies (x+4)^2=16-12\\

\implies \boxed{(x+4)^2=4}[/tex]

Answer:

(x +4)^2 = 4

Step-by-step explanation:

if we add 4 to the expression x^2 + 8x + 12 we will have a perfect square which is shown as (x +4)^2

so (x +4)^2 = 4 is equivalent to the expression x^2 + 8x + 12

The tee for the sixth hole on a golf course is 400 yards from the tee. On that hole, Marsha hooked her ball to the left, as sketched below. Find the distance between Marsha’s ball and the hole to the nearest tenth of a yard.

Answers

Answer:

181.8yd

Step-by-step explanation:

How do I even start this? And how to i order the equation to solve

Answers

[tex]f(g(h(x)))=f(g(\sqrt x))=f(\sqrt x-1)=\boxed{(\sqrt x-1)^4+4}[/tex]

This is because

[tex]h(x)=\sqrt x[/tex]

[tex]g(x)=x-1[/tex]

[tex]\implies g(h(x))=\sqrt x-1[/tex]

(that is, replace any instance of x in the definition of g with √x )

and

[tex]f(x)=x^4+4[/tex]

[tex]\implies f(\sqrt x-1)=(\sqrt x-1)^4+4[/tex]

(replace any x in f with √x - 1)

Also acceptable:

[tex](\sqrt x-1)^4+4=((\sqrt x)^4-4(\sqrt x)^3+6(\sqrt x)^2-4\sqrt x+1)+4[/tex]

[tex]=\boxed{x^2-4x\sqrt x+6x-4\sqrt x+5}[/tex]

(assuming x is not negative)

Clinton is having a coin drive. Each class has a goal of 600 pennies.Layla has already collected 338 pennies. How many more pennies does Layla need to collect to reach their goal. PLEASE SHOW YOUR WORK I WILL MARK YOU BRAINLIEST PLEASE AND EXPLAIN HOW YOU GOT YOUR ANSWER

Answers

Answer:

262 pennies

Step-by-step explanation:

If Layla already has 388 pennies and she needs 600 pennies total, all she needs to do is is take the amount she already has away from 600:

Jorge finds that 56% of his 75 classmates like salsa music and 80% of his 60 relatives like salsa music. How many more of Jorge's relatives,as compared to his classmates, like salsa music?

Answers

Answer:

6 more

Step-by-step explanation:

Find 56% of 75 first

75/x=100/56

(75/x)*x=(100/56)*x      

75=1.78571428571*x      

(1.78571428571) to get x

75/1.78571428571=x

42=x

x=42

Then Find 80% of 60

60/x=100/80

(60/x)*x=(100/80)*x    

60=1.25*x      

60/1.25=x

48=x

x=48

Subtract 48 by 42

=6

There are 120 students going in a field trip. If each bus held 30 students, how many buses are needed?
A:6
B:5
C:4
D:3

Answers

Answer:

4

Step-by-step explanation:

120/30 = 4

Since a bus can hold 30 students, that means we need 4 busses to hold all 120 students.

What x 1/4 = 2


? X 1/4 = 2

It’s an equation

Answers

the answer would be 8, since 1/4 8 times is 2
Hi! The answer would be 8 because if you do 1/4 X 8 it equals 2. I hope I explained it well enough, Goodluck.

can someone help me with this graphical method equation 3x + 5y = -2 7x - 8y = 15

Answers

X=1 and y=-1

Hope this helps :)

Answer:

x = 1

y = -1

Step-by-step explanation:

3x + 5y = -2

7x - 8y = 15

=> -8y = 15 - 7x

=> -y = 15/8 - 7/8x

=> y = -15/8 + 7/8x

3x + 5(-15/8 + 7/8x) = -2

=> 3x -75/8 + 35/8x = -2

=> 24/8x - 75/8 + 35/8x = -16/8

=> 59/8x - 75/8 = -16/8

=> 59/8x = 59/8

=> x = 59/8 x 8/59

=> x = 472/472

=> x = 1

x = 1

So, 3x + 5y = -2

=> 3 (1) + 5y = -2

=> 3 + 5y = -2

=> 5y = -5

=> y = -5/5

=> y = -1

So, x = 1

=>  y = -1

what is another expression equivalent to 3(8-2)

Answers

Answer:

3(6)

??

Step-by-step explanation:

9) Determine which sides, if any, of the figures are parallel, perpendicular, or neither.
Rectangle BACD has coordinate B(-4,-3), A(-1, -7),C(3,-4), and D(0,0).

Answers

Answer:

Parallel: AC and DB, BA and CD

Perpendicular: AC and CD, DB and CD, DB and BA, BA and AC

Neither: None

Step-by-step explanation:

Step 1: Find Slopes

Let's first find the slopes of each side of the rectangle, as that will helps us determine which sides are parallel, perpendicular, or neither.

Recall that the formula for finding the slope between two points is [tex]\frac{y_2 - y_1}{x_2-x_1}[/tex] where [tex](x_1, y_1)[/tex] and [tex](x_2,y_2)[/tex] are the coordinates of the two points. To avoid confusion, I will be taking the first point I list as [tex](x_1,y_1)[/tex] and the second point as [tex](x_2,y_2)[/tex].

Slope of [tex]BA[/tex]:

[tex]\frac{-7-(-3)}{-1-(-4)}\\=\frac{-7+3}{-1+4} \\= -\frac{4}{3}[/tex]

Slope of [tex]AC[/tex]:

[tex]\frac{-4-(-7)}{3-(-1)} \\=\frac{-4+7}{3+1} \\=\frac{3}{4}[/tex]

Slope of [tex]CD[/tex]:

[tex]\frac{0-(-4)}{0-3} \\=\frac{4}{-3} \\=-\frac{4}{3}[/tex]

Slope of [tex]DB[/tex]:

[tex]\frac{-3-0}{-4-0} \\=\frac{-3}{-4} \\=\frac{3}{4}[/tex]

Step 2: Determine which sides are parallel, perpendicular, or neither

Now that we found the slopes of the sides, we can determine which sides are parallel, perpendicular, or neither.

Recall that parallel lines have the same slope. [tex]\bf AC[/tex] and [tex]\bf DB[/tex], along with [tex]\bf BA[/tex]and [tex]\bf CD[/tex], have the same slope, so they are parallel. No other pair of sides has the same slope, so these are our only parallel pairs.

For two lines to be perpendicular, the product of their slopes must be [tex]-1[/tex]. [tex]\bf AC[/tex] and [tex]\bf CD[/tex], [tex]\bf DB[/tex] and [tex]\bf CD[/tex], [tex]\bf DB[/tex] and [tex]\bf BA[/tex], and [tex]\bf BA[/tex] and [tex]\bf AC[/tex] [tex]\bf[/tex]meet that criteria, so they are perpendicular. No other pair of sides meets the criteria, so these are our only perpendicular pairs. Hope this helps!

Find the measure of a.

Answers

Answer:

50 degrees

Step-by-step explanation:

We know that an inscribed angle in a circle is 1/2 the arc that it inscribes. So, therefore the arc is inscribed by the 25 degrees is 50. Assuming that the center of the circle is O, the center angle will be the arc measure. Knowing this, angle a is 50 degrees. If you're curious about all these theorems, they can be proved using similar triangles.

fyi, using the same logic, angle b is 25 degrees

The following diagram shows the lines x – 2y – 4 = 0, x + y = 5 and the point P(1, 1). A line is drawn from P to intersect with x – 2y – 4 = 0 at Q, and with x + y = 5 at R, so that P is the midpoint of [QR].

Find the exact coordinates of Q and of R.

Answers

Answer:

Q(2/3, -1 2/3)R(1 1/3, 3 2/3)

Step-by-step explanation:

Let (a, b) represent the coordinates of point Q on line x -2y -4 = 0. Then we know that ...

  a -2b -4 = 0

  a -2b = 4

__

If P is the midpoint of QR, then ...

  R = 2P -Q = 2(1, 1) -(a, b) = (2 -a, 2 -b)

We know this point satisfies the equation for the other line:

  x + y = 5

  (2 -a) +(2 -b) = 5

  a + b = -1 . . . . . . . . . . . rearrange to standard form

__

To find the solution to these two equations, we can subtract the first from the second:

  (a +b) -(a -2b) = (-1) -(4)

  3b = -5

  b = -5/3

  a = -1 -1b = -1 -(-5/3) = 2/3

The point Q is (a, b) = (2/3, -5/3).

The point R is ...

  R = (2 -a, 2 -b) = (2 -2/3, 2 -(-5/3)) = (4/3, 11/3)

The exact coordinates of Q and R are ...

  Q(2/3, -5/3), R(4/3, 11/3)

_____

Comment on the GeoGebra solution

The points can be found by reflecting either line across point P. Where that reflected line intersects the other line is one of the points of interest. Of course, the other is its reflection in P. You may recognize the equation for line b' (hidden in the diagram) as matching the second equation we derived above.

If you know the answer please help me

Answers

Answer:

2/7

Step-by-step explanation:

This is basically asking how many 7/10 's fit in 1/5. Or 1/5 divided by 7/10.

1/5 divided by 7/10 is the same as 1/5 times the reciprocal, 10/7. Therefore it is 1/5*10/7, or 10/35.

That simplifies to 2/7.  

ASAP!!! PLEASE

A pair of equations is shown below: y = 2x − 1 y = 4x − 5 Part A: In your own words, explain how you can solve the pair of equations graphically. Write the slope and y-intercept for each equation that you will plot on the graph to solve the equations. (6 points) Part B: What is the solution to the pair of equations? (4 points)

Answers

Step-by-step explanation:

526 133 3821

P: 12345

j _o _I _n g _I _r l

o. n z-oo-o-m

graph a
graph b
graph c
graph d

Answers

which of the following isthe√×51? answer is A .0-1

Question 3 of 10
True or false? In a two-column proof, the right column states your reasons.
A. True
OB. False
SUBMIT

Answers

Answer:

A- True

Step-by-step explanation:

If you search a picture of the graph then you will see it as well!!! Hope this helps!!!!

please i really need help The line plot below displays the fraction of incoming calls answered before the second ring by a group of employees. What fraction of employees answered less than of their incoming calls before the second ring?

Answers

Answer:

B

Step-by-step explanation:

If we count the number of points we find out that there are 36 employees

so the fraction must go like x/36

x is the numbers of dots that are less than a 1/2 which are 1/8, 1/4 and 3/8

so x=6

6/36

1/6

so option B

Find the least number which added to 3597 will make it a perfect square?

Answers

Answer:

3

Step-by-step explanation:

when 3 is added 3597 becomes 3600 which is square of 60

-n +(-3) + 3n +5
Help would be appreciated!

Ueowbfkwjdbdbdjd

Answers

Answer:

2n+2

Step-by-step explanation:

-n -3 + 3n +5

Combine like terms

-n +3n      -3 +5

2n             +2

Answer:

2n+2

Step-by-step explanation:

Maria sold t-shirts at a festival. She made 6$ for each t-shirt she sold. Her expenses were 30$. If she made a profit of 84$, how many t-shirts did she sell?

Answers

Hi! The answer is she sold 9 t shirts because she made a profit of $84 then you need to subtract $30 because that was her expenses that she already paid for which equals $54/6 = 9 so that would be 9 t sold. I hope
this helps, Goodluck.

help for abunch of points.

Answers

Answer:

It was reflected over the line x-1 or D.

Step-by-step explanation:

The triangle is no longer in the second quadrant, it moved to the 3rd quadrant meaning it had to be reflected over line x-1

Please help a brother out, I'm almost out of time

Answers

Answer:

answer is 7

Step-by-step explanation:

z-(3÷3-1)

make y = 3

z = 7

7 - (3÷3-1)

7 - 0

= 7

I hope it answers your question.

Answer: its 7

have a good day gang

Other Questions
The term crowding-out effect refers to a situation in which a government _______________ results in ______________ interest rates, causing ______________ in private spending on investment and consumer durables. Which two layers are approximately the same age?core sample 1core sample 2 Question 2 (2 points)How have new technologies helped scientists determine the age of Earth? Solve for x: 3|x + 7| = 12 x = 5 over 3, x = 19 over 3 x = 3, x = 11 x = 3, x = 11 No solution Liam tried to define a translation. - Point A maps to point A' - Every point P maps to point P' such that PP' is parallel to AA' and points in the same direction as AA' Which counterexample shows that Liam's definition does not fully define a translation? Im really confused someone explain for me!! What are the solutions to the system of equations? {y=2x28x+5y=x2 (3.5, 0.5) and (1, 1) (7, 5) and (0.5, 1.5) (3.5, 1.5) and (1, 1) (3.5, 1.5) and (1, 3) What branch can override a veto The starter motor of a car engine draws a current of 140 A from the battery. The copper wire to the motor is 4.20 mm in diameter and 1.2 m long. The starter motor runs for 0.760 s until the car engine starts.Required:a. How much charge passes through the starter motor? b. How far does an electron travel along the wire while the starter motor is on?(mm) According to John Locke, when do people have the right to revolt against their government? What is remainder when f(x) is divided by (x-a) Please help me to solve these questions as soon as possible. Thank you Which job best represents a career in the field of office administration? If a force of 1250 N acts on an area of 25 metres squared, what will be the pressure acting on the surface? Show your working as well. Enter the values needed to find the length CB Accidentally a food handler sprays glass cleaner onto a pan of baked chicken strips. Which type of hazard is this? Which sources are nonfiction narratives?Select each correct answer.a lyrical poem about daffodils in springa text explaining the history of the Civil Rights Movementa short play about talking animalsa book about the life of President John F. Kennedy Kendra, Cogley, and Mei share income and loss in a 3:2:1 ratio. The partners have decided to liquidate their partnership. On the day of liquidation their balance sheet appears as follows.KENDRA, COGLEY, AND MEIBalance SheetMay 31Assets Liabilities and Equity Cash $ 103,900 Accounts payable $ 258,000 Inventory 537,600 Kendra, Capital 76,700 Cogley, Capital 172,575 Mei, Capital 134,225 Total assets $641,500 Total liabilities and equity $641,500 Required:For each of the following scenarios, complete the schedule allocating the gain or loss on the sale of inventory. Prepare journal entries to record the below transactions. (Do not round intermediate calculations. Amounts to be deducted or Losses should be entered with a minus sign. Round your final answers to the nearest whole dollar.)(1) Inventory is sold for $608,400.(2) Inventory is sold for $469,200.(3) Inventory is sold for $358,800 and any partners with capital deficits pay in the amount of their deficits.(4) Inventory is sold for $298,800 and the partners have no assets other than those invested in the partnership.Complete this question by entering your answers in the tabs below.Required: InventoryComplete the schedule allocating the gain or loss on the sale of inventory is $608,400.1. Record the sale of inventory.2. Allocate the gain(loss) on the sale of inventory to the partners.3. Record the payment of the liabilities.4. Record the disbursement of the remaining cash to the partners. Which statement best describes the epic feature used in this excerpt and its effect on the plot? The difficult journey that Gilgamesh must undertake causes him to feel defeated. The vast landscape that Gilgamesh must travel through adds tension to the plot. The intervention of the supernatural gives Gilgamesh hope that he will defeat Humbaba. The narrative voice of the epic poem makes it difficult for the audience to relate to Gilgamesh. define micturition ?write the function of urinary system